CT5 chapter5

Discussion in 'CT5' started by Hamilton, Nov 10, 2008.

  1. Hamilton

    Hamilton Member

    The answer to question 5.9 accumulation after t years is wrong in the current notes !!
    Finding this chapter particularly tough , showing prospective reserve = retro
    is dam hard , its going to be a long evening
    anyone any advice on this
     
  2. Hamilton

    Hamilton Member

    If I thought that was bad

    Looking at the gross premium reserve tonight , its even worse . what did i spot on the papers today but thieles differential eqn , oh no says I ,it comes up .
    Why cant all the questions be like ,
    how much is an endowment worth in 3 years ? ?
     
  3. Hamilton

    Hamilton Member

    CT5 Q5 april 2007

    Just trying out loading an image and asking a question
    The Papers give the answer on the first line of image and my answer is the last line of image

    Is it the same ( baring the fact i forgot to mult by 1000 ) ? correct formula ?
    i think its the combination of the interest rate and mortality rate in one on the first line in the papers ,that is throwing me .

    is this question also like totally non real world with a mad ? mortality rate because
    the death benefit works out to be rather high at 555 considering its from birth
    the 500 quid annuity ( 5 yearly ) only works out to 879
    and the present value of all those premiums P is only 11P
    [​IMG]
     
  4. John Potter

    John Potter ActEd Tutor Staff Member

    Hamilton,

    The Question 5.9 in the 2008 CT5 notes says:

    How would the situation in the last bullet point above affect each of the retrospective and prospective reserves?

    I'm not sure this relates to your problem? Are you looking at an older set of notes?

    John
     
  5. John Potter

    John Potter ActEd Tutor Staff Member

    Hamilton

    In your answer to April 2007 Q5, I agree with your last line assuming the integral is between 0 and inifinity. You have an A bar 0 in the middle yet we're not told the person is 0 so maybe just use x? You don't really need your first line at all, the second line is a perfect start. Just in terms of exam technique, think a little bit about how your presenting your answer. You've got a sentence that starts "So" and then you write an integral - this is effectively a sentence that reads like "So, 37" Que?! :) Sounds pedantic but on a more complex qn with more calculations, the examiner might not have a clue what you're doing in any of your working and you might fail to pick up method marks that you deserved. Maybe word your answer.....

    EPV dth benefit = ...........

    = ...........

    = ........... = 555 ?

    You're dead right about the silly answer. We have a constant force of mortality for life. With the numbers in this question, my dad, who is 70, has a 2% chance of living until he is 150. I told him this and he was very pleased ;-)

    John
     
  6. Hamilton

    Hamilton Member

    Thanks for the reply

    As for the notes I have the current version for 2009 , chapter 5 question 5.9 is the accumulation after t years of a term annuity due but the answer in the notes is for a term assurance , it actually gives the exact same answer as question 5.7 .
    On to my answer , that wasn't really representative of what i would write in the exam although I probably have some bad habits from my days doing maths like not explaining anything .
    It was really a test of loading an image cause i'm going to have many questions over next few months .Dont understand why I haven't seen other people using images takes the hassle out of posting question with horrible notation in them.
    Oh yeah one more thing I really thought April 2007 q5 was from birth and judging from the post test comments of some users in a different thread other people did too.Anyway thanks for the help I'm very sure I will need plenty more.
     
    Last edited by a moderator: Nov 25, 2008
  7. Question 5.9

    Yes, Hamilton, I found the same as you! :) I thought I was maybe REALLY not understanding the work. Shouldn't the answer just be the formula directly above the question but with t's instead of n's? Or have I lost the plot totally??? :confused:
    (There's a PLOT????? :eek: )
     

Share This Page